NSG311 Exam 4

Ace your homework & exams now with Quizwiz!

Diabetic Ketoacidosis

-BG >330, high ketones, fruity breath, mental confusion, -from insufficient insulin, acute stress, etc. -untreated --> coma, which can progress to death

Shigella/Shigellosis

-Bacterial -Contaminated food or water -Incubation: 1-7 days -sick appearance, fever, fatigue and anorexia, cramps, watery or bloody diarrhea lasting 5-10 days -treatment: hydration, avoid immodium, cipro or azithro

Staphyloccoccus

-Bacterial -Inadequately cooked or refrigerated food -Incubation: 1-8hr -D/N/V -Diagnosis: ID of organism in stool, blood, food, or aspirate -Treatment: antibiotics to offending organism

Impetigo

-Bacterial Infection -Streptococcal Disease (GBS or staph aureus/MRSA) -honey colored crusts w/ local erythemia, lesions found mostly on face and extremities -Treatment: bactroban, retapamulin (altabax)

Scarlet Fever

-Bacterial infection -Streptococcal Disease Caused by group B strep. -fever, sore throat, headache, chills, rapid pulse, malaise, 12-48hr later -> sandpaper rash, inflamed tonsils w/ white exudate -treatment: PCN or amoxicillin

Obstruction of the Bile Duct (Congenital Biliary Atresia)

-Bile duct is obstructed, bile is unable to enter the intestinal tract -> accumulates in the liver -jaundice, scleral icterus, distended abdomen, prominent abdominal pain, vomiting, clay colored stool (bc bile gives it the brown color and there's no bile coming in!) -MGMT: kasai procedure, liver transplant

Lyme Disease

-Caused by borrelia burgdori (a spirochete) through tick bite -bullseye rash, heart block, stiff neck, headache, cranial nerve palsy, painful swollen arthritic joints -treatment: <8 y/o: amoxicillin >8 y/o: doxycycline

Meckel's Diverticulum

-Complication resulting from failure of the omphalomesenteric duct to fuse during embryonic development -rectal painless bleeding, abdominal pain, bloody, mucus stools -diagnostic procedures: meckel's scan

Adrenal Hormones

-Cortisol

Infectious GI Disorders with bloody diarrhea

-E.Coli (starts as watery) -Salmonella -Shigella (can be either bloody or watery)

Bladder Exstrophy

-Exposed bladder, urethra, and ureteral orfices through the suprapubic area

Enterbius Vermicularis (pinworm)

-Helminthic Infection (not bacterial or viral bc it's a WORM) -fecal-oral ingested or inhaled eggs that hatch in the upper intestine and mature. after mating, worms migrate out of the intestine and lay eggs -perianal itching, enuresis, sleeplessness, restlessness, and irritability due to itching -diagnosis: tape test -treatment: mebendazole, albendazole, and pyrantel pamoate

Celiac Disease

-Immune-mediated abnormal response to gluten -steatorrhea, ab distention, pain, failure to thrive -usually coincides w/ T1DM -Diagnosis: endoscopy w biopsy of villi in small intestines (GOLD STANDARD) -Treatment: diet alterations

Appendicitis

-Inflammation caused by an obstruction of the lumen of the appendix -RLQ abdominal rebound tenderness, rigid abdomen, fever, diarrhea or constipation, vomiting -Diagnosis: CT (gold standard) -surgery needed

IBD: Chron's Disease

-Inflammation from mouth to anus -abdominal pain, diarrhea w or w/o blood, weight loss. risk for fistulas -treatment: immune modulators, surgery may be needed but it's not curative.

Acetaminophen Poisoning

-Liver involvement -N/V, sweating, pallor, PAIN IN RUQ, CONFUSION, jaundice, stupor, coagulation -treatment: N-ACETYLCYSTEINE (MUCOMYST) / ACTIVATED CHARCOAL

Gastroesophageal Reflux (GER)

-NOT GERD -Gastric content reflux back up into the esophagus in infants. Usually self limiting and resolves by 1 year of age -infants: spitting up/forceful vomiting, irritability, excessive crying, blood in vomit, respiratory problems -children: heartburn, ab pain, difficulty swallowing, chronic cough, noncardiac chest pain -diagnosis: upper gi probe, endoscopy -MGMT: small meals, sit up, burp before and after feeding, thicken breastmilk/formula

Phimosis

-Narrowing of the preputial opening of the foreskin, resulting in an inability to retract the foreskin of the penis. -NORMAL FINDING in infants and young boys and usually disappears as the child grows -DONT RETRACT FORESKIN

Epispadias

-Opening of the urinary meatus on the dorsal or superior surface of the penis (TOP PART) -mgmt: surgery, topical testosterone or injections -may have associated chordee (fibrous band pulling the penis downward) or cryptorchidism (undescended testicles)

Minimal Change Nephrotic Syndrome (MCNS)

-Peak incidence between 2-3 years of age -cause is unknown, but it can have a multifactorial etiology -same symptoms as nephrotic syndrome

Intussusception

-Proximal segment of the bowel telescopes into a more distal segment. -Sudden episodic ab pain, screaming pain, stools that look like red currant jelly, vomiting, fever, tender, distended abdomen -Management: air enema or surgical repair

Infectious GI Disorders with Watery Diarrhea

-Rotavirus -E.Coli (progresses to bloody) -C.Diff -Shigella (can be either bloody or watery)

Hirschsprung's Disease (Congenital Aganglionic Megacolon)

-Structural anomaly of the GI tract caused by lack of ganglionic cells in segments of the colon -Hirschsprung's means no dung -FAILURE TO PASS MECONIUM WITHIN 24-48HR AFTER BIRTH, vomiting bile (bc bile gets emptied into the duodenum, which is obstructed in this case), ab distention, refusal to eat -Treatment: prepare for surgery -Complications: Short gut syndrome

Thyroid Hormones

-T3 -T4

Hypertrophic Pyloric Stenosis

-Thickening of the pyloric sphincter, which creates an obstruction. Usually occurs the first few weeks of life. -Exorcist vomiting, blood tinged vomit, constant hunger, olive shaped mass in RUQ, dry membranes, failure to gain weight. Just can't hold anything down. -MGMT: NG tube, NPO, IVF, strict I&O

Fungal Infections

-Tinea cruris (jock itch) -Tinea pedis (athletes foot) -Tinea capitis (scalp) -Tinea corporis (body) THIS IS ALL RINGWORM Treatment: nystatin, clotrimazole, miconazole, etc.

Testicular or Ovarian Torsion

-Twisting of the male or female sex organs leading to lack or loss of blood flow and potential necrosis -SURGICAL EMERGENCY to potentially save the reproductive organs -diagnosis: immediate ultrasound

Hypospadias

-Urethral defect in which the urethral opening is not at the end of the penis but on the ventral (LOWER) aspect of the penis -mgmt: surgery, topical testosterone or injections -may have associated chordee (fibrous band pulling the penis downward) or cryptorchidism (undescended testicles)

Measles (Rubeola)

-Viral Infection -Airborne Precautions -Before rash (3-4 days): mild to high fever, conjunctivitis, fatigue, cough, runny nose, sore throat, koplik spots, enlarged lymph nodes -Rash: first appears on face and back and then quickly spreads to whole body -Treatment antipyretics, antihistamines

Chickenpox (Varicella)

-Viral Infection -Airborne Precautions -Before rash (1-2 days): fever, fatigue, loss of appetite, headache -Rash: macules start in center of trunk, spreading from the face and proximal extremities macules --> papules -> vesicles -> crust -> scabs -Treatment: tyelenol, motrin, calamine lotion

Infectious Mononucleosis (Mono)

-Viral Infection -Contact Precautions spread by saliva. healthy ppl can carry epstein barr virus, which starts mono. Before rash (1 week): cold symptoms, increased WBC, splenomegaly Rash: maculopapular red spots symmetrically distributed on upper and lower extremities, progressing proximal to distal surfaces through 1 week

Hemolytic Uremic Syndrome

-acute renal failure, hemolytic anemia, and thrombocytopenia. Toxins enter the bloodstream and destroy RBCs -> Breakdown of rbcs clogs the kidneys -COMMONLY CAUSED BY E.COLI -diarrhea positive and diarrhea negative HUS -D/V/, bruising, purpura, anuric and hypertensive in severe form -mgmt: anti-hypertensives, blood transfusions

Nephrotic Syndrome

-alterations in the glomerular membrane allow proteins (especially albumin) to pass into the urine, resulting in decreased blood osmotic pressure, which leads to proteinuria, hyperlipidemia, and edema -weight gain, facial and periorbital edema (decreases throughout day), ascities, edema to lower extremities and genitalia, decreased frothy urine -mgmt:, high dose steroids, diuretics albumin and lasix

Pituitary Hormones

-antidiuretic hormone (ADH) -corticotropic (ACTH) -somatropin (GH) -Thyrotropin (TSH)

Epidural Hematoma/Bleed

-arterial bleeding into the space between the dura and the skull -LOC WITH INJURY, UNEQUAL DILATION OR CONSTRICTION OF THE PUPILS, DECORTICATE POSTURING, vomiting, headache, seizures, or hemiparesis -mgmt: surgical removal of the accumulated blood and cauterization or ligation of the torn artery

Head Injury Treatment

-assess pupils, LOC for increased ICP -symptoms in infants: bulging fontanel, cranial suture separation, high pitched cry, increased head circumference -hypertonic saline, mannitol, dexamethasone, rhinorhea/otorrhea treatment

Salmonella

-bacteria -person to person, undercooked meats & poultry -incubation period: 6-72 hr -mild-severe nausea, v, abdominal cramping, BLOODY DIARRHEA, fever, headache, confusion -can lead to meningitis or septicemia -treatment: cipro or azithromycin

Clostridium Difficile (C Diff)

-bacterial -contact w/ colonized spores. can occur from overgrowth of c.diff following antibiotic therapy. -mild, WATERY diarrhea, leukocytosis, hypoalbuminemia, high fever. FOUL SMELLING POOP. -treatment: flagyl, vanco, donor stool replacement

Escheria Coli

-bacterial -usually foodborne -incubation period: 3-4 days -watery diarrhea (1-2) -> BLOODY, abdominal cramping -could lead to hemolytic uremic syndrome -treatment: self limiting so no antibiotics needed. No immodium (an anti-diarrheal) because you want them to poop out all the bacteria.

congenital heart disorders with obstruction to blood flow

-coarctation of the aorta -pulmonary stenosis -aortic stenosis

Acute bilirubin encephalopathy

-correlates w/ hemolytic disease (hyperbilirubinemia) -destruction of brain cells d/t invasion of indirect, unconjugated bilirubin. Caused by excessive breakdown of RBC at birth. common in preterm.

Lead Poisoning

-distractibility, impulsiveness, hyperactivity, hearing impairment, mild intellectual disability (low dose) -cognitive delays varying in severity, blindness, paralysis, coma, seizures, and death (high dose) -CHELATION THERAPY w calcium disodium versenate

Cushing's Triad

-for head injury 1. bradycardia 2. irregular breathing 3. hypertension

Irritable Bowel Syndrome

-functional bowel disorder that typically causes symptoms of abdominal pain and altered bowel habits with no underlying organic cause. -May follow an infection that may have altered the intestinal flora -sensitive gut basically

Hydrocarbon Poisoning

-gasoline, kerosene, lighter fluid, paint thinner, turpentine poisoning -gagging, choking, coughing, N/V, lethargy, weakness, tachypnea, cyanosis, grunting, retractions -DO NOT INDUCE VOMITING (IRRITATES THROAT), intubation with endotracheal tube, gastric decontamination, steroids to treat chemical pneumonitis

Corrosives Poisoning

-household cleaning, batteries, denture cleaners, bleach -pain and burning in mouth, throat, and stomach, edematous lips, tongue, and pharynx, violent vomiting -DO NOT INDUCE VOMITING, airway maintenance, NPO, do not attempt to neutralize acid, analgesics

Constipation

-inability to have a bowel movement -abdominal distention, foul smelling gas, abdominal pain, vomiting, palpable stool -diagnosis: x-ray -MGMT: polyethylene glycol, fiber intake, miralax (draws fluid to GI tract and rehydrates hard stool)

Acute Glomerulonephritis

-inflammation of the glomeruli of the kidney, common between ages 5-10 -in most children the course is limited, short-term and benign -some cases will continue to chronic nephritis -post strep pharyngitis or impetigo in the last 7-21 days, tea colored urine, edema, V/A, edema, low-grade fever -mgmt: antibiotics, diuretics, KAYEXALATE THERAPY (binds to potassium), diuretics, anti-hypertensives

IBD: Ulcerative Colitis

-inflammation only in the colon and rectum -crampy abdominal pain, urgency, tenesmus, frequent bloody stools, anemia and hypoalbuminemia can happen -treatment: immune modulators. surgery may be needed and it's curative

Acetylsalicylic Acid (Aspirin) Poisoning

-n/v, disorientation, diaphoresis, tachypnea, tinnitus, oliguria, lightheadedness, seizures, bleeding tendencies, dehydration, seizures -ACTIVATED CHARCOAL, GASTRIC LAVAGE, ***VITAMIN K INJECTION (ANTIDOTE)***, HEMODIALYSIS IN SEVERE CASES, NaCO3, oxygen/vent

Giardia Lamblia

-parasitic pathogen -person to person, food, animals -children <5: D/V, anorexia -children >5: ab cramps, intermittent loose stools, malodorous pale greasy stools -treatment: metronidazole and tinidazole

Enuresis

-repeated involuntary or intentional urination, with no identified organic cause -inappropriate urination during the day or night at least twice a week for at least 3 months -primary: child has never been free of bed wetting for any extended periods of time -secondary: child who started bed wetting after development of urinary control (most common after sexual abuse)

Urinary Tract Infections

-risk factors: urinary stasis, urinary tract anomalies, constipation, female, bad hygeine, sex -mgmt: antibiotics

B Hemolytic Group B Strep Infection

-risk: prolonged ROM -gbs is a normal inhabitant of female genital tract -complication: meningitis (lethargy, loss of appetite, bulging fontanelles, increased ICP) -treatment: PCN, cefazolin, clindamycin, vanco

Burn Management

-silver sulfadiazine and bactracin -escharotomy (surgical removal of dead tissue that surrounds the burn) -debridement (sterile procedure where dead tissue is scrubbed off) -allografting (cadaver) -autogradting (from patient)

congenital heart disorders with decreased pulmonary blood flow

-tetralogy of fallot -tricuspid atresia

Chronic Kidney Disease

-the progressive loss of renal function over months or years -becomes chronic (after acute), causing permanent nephron destruction -fatigue on exertion, delayed growth, uremic odor to breathe, muscle cramps and headaches, bruised skin, amenorrhea -mgmt: kidney biopsy, SURGICAL TRANSPLANT NEEDED

Subdural Hematoma/Bleed

-venous bleeding into the space between the dura and arachnoid membrane -USUALLY DO NOT HAVE LOC, sz, bleeding, vomiting, hyperirritability, increased head circ, closed sutures and fontanel -mgmt: needle aspiration of accumulated blood, allowing time for venous tear to clot

Congenital heart disorders with increased pulmonary blood flow

-ventricular septal defect (VSD) -atrial septal defect (ASD) -patent ductus arteriosus (PDA)

Rotavirus

-viral -fecal oral transmission -incubation period: 48hr -fever, WATERY STOOLS, diarrhea, vomiting -treatment: supportive care, rotavirus vaccine

Erythema Infectiosum (Fifth Disease)

-viral infection -caused by parvo virus -Before rash (several days): fever, runny nose, headache -Rash: SLAP CHEEK DISEASE -Treatment: analgesics, antipyretics, antihistamines

Supplemental Iron Overdose / Poisoning

-vomiting, bloody emesis, diarrhea, gastric pain, bloody stools, metabolic acidosis, hyperglycemia, bleeding, fever, shock, and possible death -CHELATION THERAPY using deferoxamine mesylate (desferal)

Skull Fracture

-vomiting, ecchymosis around the eyes or behind the ears (racoon eyes and battle's sign) -irritability, evere headache, photophobia -mgmt: skull removal/plate and screw placement

Rheumatic Fever

A bacterial infection that can be carried in the blood to the joints. Approx 10 days after recovery from pharyngitis -fever, tachycardia, cardiomegaly, new or changed heart murmur, muffled heart sounds, pericardial friction rub, large joints, erythema marginatrum, chorea -treatment: pennicillin therapy

Aortic Stenosis

A narrowing of the aortic valve -obstruction of blood flow -faint pulses, hypotension, tachycardia, intolerance to exercise, dizziness, murmur -treatment: nonsurgical (ballon dilation w/ cardiac cath, beta blockers, calcium channel blockers), surgical (norwood procedure, aortic valvotomy)

Apparent life-threatening event (ALTE)

A sudden event in infants under the age of 1 year, during which the infant experiences a combination of symptoms including apnea, change in color, change in muscle tone and coughing or gagging. -different from apnea bc no breathing doesn't have to be longer than 20 seconds.

The physician of a child with extensive burns tells the nurse an hour after the injury to "monitor him carefully." Which of the following would be the most important factor to monitor closely? A. Urinary output B. Pupillary reaction C. Intracranial pressure D. Sugar and acetone of urine

A.

A nurse is assessing a client who has pertussis. Which of the following are clinical manifestations of pertussis? (Select all that apply.) A. Runny nose B. Mild fever C. Whooping sound cough D. Swollen salivary glands E. Red rash

A. A client who has pertussis has cold like symptoms, including runny nose, congestion, and mild fever. B. A client who has pertussis has cold like symptoms, including runny nose, congestion, and mild fever. C. A client who has pertussis will experience coughing fits and a whooping sound.

A nurse is teaching a group of caregivers about E. coli. Which of the following information should the nurse include in the teaching? (Select all that apply.) A. Abdominal cramping occurs. B. Watery diarrhea is present for more than 5 days. C. It can lead to hemolytic uremic syndrome. D. It is a foodborne pathogen. E. Antibiotics are given for treatment

A. Abdominal cramping occurs. C. It can lead to hemolytic uremic syndrome. D. It is a foodborne pathogen.

A nurse is assessing a child with short stature. Which of the following findings would indicate a growth hormone deficiency? A. Proportional height to weight B. Height proportionally greater than weight C. Weight proportionally greater than height D. BMI greater than height/weight ratio

A. CORRECT": Children who have growth hormone deficiency present with short stature with proportional height and weight.

A nurse is caring for a child who has short stature. Which of the following diagnostic tests should be completed to confirm growth hormone (GH) deficiency? (Select all that apply.) A. CT scan of the head B. Bone age scan C. GH stimulation test D. Serum IGF-1E. DNA testing

A. CORRECT: A CT scan of the head is conducted to determine whether there is a structural component to the short stature. B. CORRECT: A bone age scan is conducted to determine the development of the bones. C. CORRECT: A GH stimulation test is conducted to confirm diagnosis of GH deficiency. D. CORRECT: A serum IGF-1 is obtained as a preliminary test to determine GH deficiency.

A nurse is assessing an infant who has hypertrophic pyloric stenosis. Which of the following findings should the nurse expect? (Select all that apply.) A. Projectile vomiting B. Dry mucus membranes C. Currant jelly stools D. Sausage‑shaped abdominal mass E. Constant hunger

A. CORRECT: A client who has a pyloric stricture has thickening of the pyloric sphincter, resulting in projectile vomiting. B. CORRECT: A client who has pyloric stricture is unable to consume adequate food and fluid, resulting in dehydration. Dry mucous membranes is a manifestation of hypertrophic pyloric stenosis. E. CORRECT: A client who has pyloric stricture is unable to consume adequate food and fluid, resulting in constant hunger.

A nurse is reviewing sick day management with a parent of a child who has type 1 diabetes mellitus. Which of the following should the nurse include in the teaching? (Select all that apply.) A. Monitor blood glucose levels every 3 hr. B. Discontinue taking insulin until feeling better. C. Drink 8 oz of fruit juice every hour. D. Test urine for ketones.E. Call the health care provider if blood glucose is greater than 240 mg/dL.

A. CORRECT: A client who is experiencing illness can have waning blood glucose levels. Frequent monitoring of blood glucose levels is done to identify hyperglycemic or hypoglycemic episodes. D. CORRECT: A client who is experiencing an illness should test her urine for ketones to assist in early detection of ketoacidosis. E. CORRECT: A client who is experiencing illness should notify the provider of blood glucose levels greater than 240 mg/dL to obtain further instructions in caring for the hyperglycemia.

A nurse is teaching the parent of a child who has a growth hormone deficiency. Which of the following are complications of untreated growth hormone deficiency? (Select all that apply.) A. Delayed sexual development B. Premature aging C. Advanced bone age D. Short stature E. Increased epiphyseal closure

A. CORRECT: A complication of untreated growth hormone deficiency includes delayed sexual development. B. CORRECT: A complication of untreated growth hormone deficiency includes premature aging. D. CORRECT: A complication of untreated growth hormone deficiency includes short stature.

A nurse is caring for a child who is suspected to have Enterobius vermicularis. Which of the following actions should the nurse take? A. Perform a tape test. B. Collect stool specimen for culture. C. Test the stool for occult blood. D. Initiate IV fluids.

A. CORRECT: A tape test is used when diagnosing Enterobius vermicularis.

A nurse is caring for a child who has Meckel's diverticulum. Which of the following manifestations should the nurse expect? (Select all that apply.) A. Abdominal pain B. Fever C. Mucus, bloody stools D. Vomiting E. Rapid, shallow breathing

A. CORRECT: Abdominal pain is a manifestation of Meckel's diverticulum. C. CORRECT: Mucus and bloody stools are a manifestation of Meckel's diverticulum.

A nurse is caring for a male infant who has an epispadias. Which of the following findings should the nurse expect?(Select all that apply.) A. Bladder exstrophy B. Inability to retract foreskin C. Widened pubic symphysis D. Broad, spade-like penisE. Pain

A. CORRECT: Bladder exstrophy is a manifestationof a male infant who has epispadias. C. CORRECT: Widened pubic symphysis is a manifestation of a male infant who has epispadias. D. CORRECT: Broad, spade-like penis is a manifestation of a male infant who has epispadias.

A nurse is assessing a child who has a rotavirus infection. Which of the following are expected findings? (Select all that apply.) A. Fever B. Vomiting C. Watery stools D. Bloody stools E. Confusion

A. CORRECT: Fever is a manifestation of rotavirus infection. B. CORRECT: Vomiting for approximately 2 days is a manifestation of rotavirus infection. C. CORRECT: Foul‑smelling, watery stools is a manifestation of rotavirus infection.

A nurse is teaching a parent of an infant about gastrointestinal reflux disease. Which of the following should the nurse include in the teaching? (Select all that apply.) A. Offer frequent feedings. B. Thicken formula with rice cereal. C. Use a bottle with a one‑way valve. D. Position baby upright after feedings. E. Use a wide‑based nipple for feedings.

A. CORRECT: Frequent feeding will assist in decreasing the amount of vomiting episodes. B. CORRECT: Thickened formula will assist in decreasing the amount of vomiting episodes. D. CORRECT: Positioning the infant in an upright position following feedings will assist in decreasing the amount of vomiting episodes.

A nurse is caring for an infant who has ambiguous genitalia. Which of the following actions should the nurse take? (Select all that apply.) A. Prepare the child for surgery. B. Obtain a detailed family history. C. Gather supplies for a circumcision. D. Refer the family for genetic counseling. E. Explain the need for a chromosomal analysis.

A. CORRECT: Infants who have ambiguous genitalia will need surgery. Preparing the family for surgery is an appropriate action for the nurse to take. B. CORRECT: A detailed family history is used for gender assignment, therefore an appropriate action for the nurse to take. D. CORRECT: Families with an infant who has ambiguous genitalia will need ongoing support. Referring to genetic counseling is an appropriate action for the nurse to take. E. CORRECT: Chromosomal analysis is used for gender assignment, therefore an appropriate action for the nurse to take.

A nurse is admitting a child who has severely symptomatic HIV. Which of the following findings should the nurse expect? (Select all that apply.) A. Kaposi's sarcoma B. Hepatitis C. Wasting syndrome D. Pulmonary candidiasis E. Cardiomyopathy

A. CORRECT: Kaposi's sarcoma is a manifestationof a child who is severely symptomatic. C. CORRECT: Wasting syndrome is a manifestationof a child who is severely symptomatic. D. CORRECT: Pulmonary candidiasis is a manifestation of a child who is severely symptomatic.

A nurse is teaching a parent of a child who has HIV. Which of the following information should the nurse include? (Select all that apply.) A. Obtain yearly influenza vaccination. B. Monitor a fever for 24 hr before seeking medical care. C. Avoid individuals who have colds. D. Provide nutritional supplements. E. Administer aspirin for pain.

A. CORRECT: Obtaining a yearly influenza vaccination is recommended to protect the child from opportunistic infections. C. CORRECT: Avoiding individuals who have colds will assist in protecting the child from opportunistic infections. D. CORRECT: Nutritional supplements are recommended to promote improved nutrition of the child who has HIV.

A nurse is caring for a 10‑year‑old child who has nephroticsyndrome. Which of the following findings should the nurse report to the provider? A. Serum protein 5.0 g/dL B. Hgb 14.5 g/dL C. Hct 40% D. Platelet 200,000 mm3

A. CORRECT: Serum protein 5.0 g/dL is out of the expected reference range for a 10‑year‑old child and should be reported to the provider.

A nurse is teaching a group of parents about E. coli. Which of the following information should the nurse include in the teaching? (Select all that apply.) A. Severe abdominal cramping occurs. B. Watery diarrhea is present for more than 5 days. C. It can lead to hemolytic uremic syndrome. D. It is a foodborne pathogen. E. Antibiotics are given for treatment.

A. CORRECT: Severe abdominal cramping is a manifestation of E. coli. C. CORRECT: E. coli can lead to hemolytic uremic syndrome. D. CORRECT: E. coli is a foodborne pathogen.

A nurse is teaching the parent of a child who has hand, foot, and mouth disease. Which of the following should be included in the teaching? A. "Your child can be contagious when the symptoms are gone." B. "The incubation period is 10 to 21 days." C. "It is transmitted by droplet." D. "Once infected, your child will be a lifetime carrier."

A. Children are most contagious the first week of illness. However, they can be contagious even when symptoms are gone.

A nurse is teaching a parent about post traumatic stress disorder (PTSD). Which of the following should be included in the teaching? (Select all that apply.) A. Children who have PTSD require psychotherapy. B. A clinical manifestation of PTSD is phobias. C. Depression is seen within 1 day after the incident. D. PTSD develops following a traumatic event. E. There are six stages of PTSD.

A. Children who have PTSD should be referred to psychotherapy to assist with resolution of the traumatic event. B. The child who is experiencing PTSD often has new phobias that can be related to the traumatic event. D. PTSD develops following a traumatic event such as assault, serious injury, or a life‑threatening episode.

A nurse is planning to administer immunizations to a 4-year-old child who has up-to-date immunizations. Which of the following should the nurse anticipate giving? (Select all that apply.) A. Inactivated poliovirus (IPV) B. Haemophilus influenzae type b (Hib) C. Measles, mumps, rubella (MMR) D. Varicella (VAR) E. Hepatitis B (HepB) F. Diphtheria, tetanus, pertussis (DTaP)

A. Four doses of inactivated poliovirus vaccine are given during childhood with a dosegiven at 4 years of age. C. Two doses of measles, mumps, and rubella vaccine are given during childhood with adose given at 4 years of age. D. Two doses of varicella vaccine are given during childhood with a dose given at 4 years of age. F. Six doses of diphtheria, tetanus, pertussis vaccine are given during childhood with a dose given at 4 years of age.

A nurse is caring for a client who has a skin graft. Which of the following clinical manifestations indicate infection? (Select all that apply.) A. Green color to subcutaneous fat B. Unstable body temperature C. Generation of granulation tissue D. Subeschar hemorrhage E. Change in skin color around the affected area

A. Green color to subcutaneous fat is a clinical manifestation of infection. B. Unstable body temperature is a clinical manifestation of infection. D. Subeschar hemorrhage is a clinical manifestation of infection. E. A discoloration of the skin around the burn is a clinical manifestation of infection.

A nurse is caring for a client who has major burns and suspected septic shock. Which of the following findings are consistent with septic shock? (Select all that apply.) A. Increased body temperatureB. Altered sensorium C. Decreased capillary refill D. Decreased urine output E. Increased bowel sounds

A. Increased body temperature is a clinical manifestation of septic shock. B. Altered sensorium is a clinical manifestation of septic shock. D. Decreased urine output is a clinical manifestation of septic shock.

A nurse is caring for a client who has a major burn and is experiencing severe pain. Which of the following is an appropriate nursing intervention to manage this client's pain? A. Administer morphine sulfate IV via continuous infusion. B. Administer meperidine (Demerol) IM as needed. C. Administer acetaminophen (Tylenol) PO every 4 hr. D. Administer hydrocodone (Vicodin) PO every 6 hr.

A. Opioids administered IV via continuous infusion are recommended for clients who have major burns.

Which of the following situations is likely to precipitate a hypoglycemic reaction in a child with diabetes mellitus? A. Participation in a soccer game B. Eating a high-carbohydrate lunch C. Developing an upper respiratory infection D. Forgetting to take insulin

A. Participation in a soccer game

A nurse is assessing an infant who has hypertrophic pyloric stenosis. Which of the following manifestations should the nurse expect? (Select all that apply.) A. Projectile vomiting B. Dry mucus membranes C. Currant jelly stools D. Sausage-shaped abdominal mass E. Constant hunger

A. Projectile vomiting B. Dry mucus membranes E. Constant hunger

Parents are told that their infant has a heart defect with a left-to-right shunt. What is the best way for the nurse to explain this type of shunting to the parents? A. This type of shunting causes an increase of blood to the lungs. B. This type of shunting causes an increase of blood to the systemic circulation. C. This type of shunting causes a decrease of blood to the lungs. D. This type of shunting causes a decrease of blood to the brain.

A. This type of shunting causes an increase of blood to the lungs.

A nurse is planning to administer immunizations to a 2-month-old infant. Which of the following should the nurse anticipate giving? (Select all that apply.) A. Rotavirus (RV) B. Diphtheria, tetanus, pertussis (DTaP) C. Haemophilus influenzae type b (Hib) D. Hepatitis A (HepA) E. Pneumococcal (PCV) F. Inactivated poliovirus (IPV)

A. Two doses of rotavirus vaccine are given during childhood starting at 2 months of age. B. Six doses of diphtheria, tetanus, pertussis vaccine are given during childhood starting at2 months of age. C. Three doses of haemophilus influenzae type b vaccine are given during childhood starting at 2 months of age. E. Four doses of pneumococcal vaccine are given during childhood starting at 2 months of age. F. Four doses of inactivated poliovirus vaccine are given during childhood starting at2 months of age.

A nurse is assessing an infant who has coarctation of the aorta. Which of the following findings should the nurse expect? SATA. A. Weak femoral pulses B. Cool skin of lower extremities C. Severe cyanosis D. Clubbing of the fingers E. Low blood pressure

A. Weak femoral pulses B. Cool skin of lower extremities E. Low blood pressure

A nurse is assessing an infant who has coarctation of the aorta. Which of the following findings should the nurse expect? Select all that apply. A. Weak femoral pulses B. Warm skin of lower extremities C. Severe cyanosis D. Clubbing of the fingers E. High blood pressure in right upper extremity

A. Weak femoral pulses E. High blood pressure in right upper extremity

During the newborn examination, the nurse would suspect spina bifida occulta if what finding is present? Select all that apply. A. a dimpling at the base of spine B. abnormal tufts of hair at the base of spine C. discolored skin at the base of spine D. head circumference above the 90th percentile E. continuous dribbling of urine

A. a dimpling at the base of spine B. abnormal tufts of hair at the base of spine C. discolored skin at the base of spine

A nurse is caring for a child who is suspected of having rheumatic fever. Which of the following findings should the nurse expect? (select all that apply) A. erythema marginatum (rash) B. continuous joint pain of the digits C. Tender, subq nodules D. Decreased erythrocyte sedimentation rate E. Elevated C-Reactive protein

A. erythema marginatum (rash) E. Elevated C-Reactive protein

What is a classic sign of neonatal respiratory distress syndrome? Select all that apply. A. expiratory grunting B. nasal flaring C. retractions D. tachypnea E. bradypnea

A. expiratory grunting B. nasal flaring C. retractions D. tachypnea

While caring for a newborn with cleft lip, the nurse would assess which activity that may be compromised due to the cleft lip? A. sucking ability B. respiratory status C. locomotion D. GI function

A. sucking ability

Scarlet fever is a childhood exanthema. Which is the causative agent? A.Streptococci B.Staphylococci C.An equinovarus D.A syncytial virus

A.Streptococci

Suppose Maia's baby develops torticollis, and she is distraught by her baby's appearance. What care measure would best relieve the infant's physical anomaly and Maia's distress? A.Teach Maia how to perform her baby's neck stretching exercises. B.Wrap the infant's neck in a warm towel for 15 minutes twice daily. C.Assure Maia that the anomaly will resolve spontaneously. D.Administer 80 mg of aspirin with each of his bottle feedings.

A.Teach Maia how to perform her baby's neck stretching exercises.

A nurse is caring for a client who has a major burn and is experiencing severe pain. Which of the following actions should the nurse implement to manage this client's pain? A. Administer morphine sulfate IV via continuous infusion. B. Administer meperidine IM as needed. C. Administer acetaminophen PO every 4 hr. D. Administer hydrocodone PO every 6 hr.

A.morphine

Contact Dermatitis

An inflammation of the skin caused by having contact with certain chemicals or substances

A nurse is caring for a client who has a skin graft. Which of the following manifestations indicate infection? (Select all that apply.) A. Pink color to subcutaneous fat B. Unstable body temperature C. Generation of granulation tissue D. Hyperventilation E. Change in skin color around the affected area

B,D,E

The nurse is teaching an in-service program to a group of nurses on the topic of children diagnosed with rheumatic fever. The nurses in the group make the following statements. Which statement is most accurate regarding the diagnosis of rheumatic fever? A. "This disorder is caused by genetic factors." B. "Children who have this diagnosis may have had strep throat." C. "Being up-to-date on immunizations is the best way to prevent this disorder." D. "The onset and progression of this disorder is rapid."

B. "Children who have this diagnosis may have had strep throat."

A nurse is providing instruction to the teacher of a child who has attention-deficit/hyperactivity disorder (ADHD). Which of the following classroom strategies should be included in the teaching?(Select all that apply.) A. Eliminate testing. B. Allow for regular breaks. C. Combine verbal instruction with visual cues. D. Establish consistent classroom rules. E. Decrease stimuli in the environment.

B. Allowing for regular breaks will assist the client who has ADHD to focus on the required tasks. C. Combining verbal instruction with visual cues will assist the client who has ADHD with learning information. D. Providing consistent classroom rules will assist the client who has ADHD to become successful. E. Stimuli in the environment distract the client who has ADHD, so they should be decreased.

A nurse is caring for a client who has a superficial partial thickness burn. Which of the following is an appropriate action for the nurse to take? A. Administer an IV infusion of 0.9% sodium chloride. B. Apply cool, wet compresses to affected area. C. Clean the affected area using a soft-bristle brush. D. Administer morphine sulfate.

B. Applying cool, wet compresses stops the burn process. Therefore, this is an appropriate action for the nurse to take.

A nurse is caring for a child who has post streptococcal glomerulonephritis(APSGN). Which of the following manifestations should the nurse expect? (Select all that apply.) A. Frothy urine B. Periorbital edema C. Ill appearance D. Decreased creatinine E. Hypertension

B. CORRECT: A client who has APSGN will exhibit periorbital edema due to decrease in plasma filtration. C. CORRECT: A client who has APSGN will exhibit an ill appearance due to the manifestations experienced from the inadequate functioning of the kidneys. E. CORRECT: A client who has APSGN will exhibit hypertension due to inadequate function of the kidneys and possibly edema.

A nurse is caring for a child who has Hirschsprung's disease.Which of the following actions should the nurse take? A. Encourage a high‑fiber, low‑protein, low‑calorie diet. B. Prepare the family for surgery. C. Place an NG tube for decompression. D. Initiate bed rest.

B. CORRECT: A client who has Hirschsprung's disease requires surgery to remove the affected segment of the intestine. Preparing the family for surgery is an appropriate action for the nurse to take.

A nurse is providing preconception teaching with a client who has phenylketonuria (PKU). Which of the following information should the nurse include in the teaching? A. Follow a low‑phenylalanine diet once pregnancy is confirmed. B. The client will undergo testing of phenylalanine levels one to two times per week throughout pregnancy. C. Increase intake of dietary proteins prior to conception. D. The client will require a cesarean section birth due to the likelihood of having a fetus with macrosomia.

B. CORRECT: A client who has PKU will have her phenylalanine levels monitored one to two times per week throughout pregnancy.

A nurse is assessing a child who has nephrotic syndrome. Which of the following findings should the nurse expect?(Select all that apply.) A. Dipstick protein of 1+ B. Edema in the ankles C. Hyperlipidemia D. Weight lossE. Anorexia

B. CORRECT: A client who has nephrotic syndrome will exhibit edema in the ankles due to the decreasing colloidal osmotic pressure in the capillaries. C. CORRECT: A client who has nephrotic syndrome will exhibit hyperlipidemia due to the increased hepatic synthesis of proteins and lipids. E. CORRECT: A client who has nephrotic syndrome will exhibit anorexia due to the edema of the intestinal mucosa.

A nurse is assessing a newborn who has congenital hypothyroidism. Which of the following findings should the nurse expect? (Select all that apply.) A. Hypertonicity B. Cool extremities C. Short neck D. Tachycardia E. Hyperreflexia

B. CORRECT: Cool extremities are an expected finding in a newborn who has congenital hypothyroidism. C. CORRECT: A short neck is an expected finding in a newborn who has congenital hypothyroidism.

A nurse is teaching a group of adolescents about HIV/AIDS. Which of the following statements should the nurse include in the teaching? A. "You can contract HIV through casual kissing." B. "HIV is transmitted through IV substance use." C. "HIV is now curable if caught in the early stages." D. "Medications inhibit transmission of the HIV virus."

B. CORRECT: HIV is transmitted via blood, semen, vaginal secretions and breast milk. IV substance use is a potential mode of transmission.

A nurse is teaching an adolescent who has diabetes about clinical manifestations of hypoglycemia. Which of the following should be included int he teaching? (Select all that apply.) A. Increased urination B. Hunger C. Signs of dehydration D. Irritability E. Sweating and pallor F. Kussmaul respirations

B. CORRECT: Hunger is a manifestation of hypoglycemia because of the increased adrenergic nervous system activity. D. CORRECT: Irritability is a manifestation of hypoglycemia because of the depleted glucose in the CNS. E. CORRECT: Sweating and pallor is a manifestation of hypoglycemia because of the increased adrenergic nervous system activity.

A nurse is caring for an infant who has a hydrocele. Which of the following actions should the nurse take? A. Prepare the child for surgery. B. Explain to the parents that the issue will self-resolve. C. Retract the foreskin and cleanse several times daily. D. Refer the family for genetic counseling.

B. CORRECT: Hydrocele is fluid in the scrotum and resolves spontaneously in the majority of cases.

A nurse is caring for a child who has had watery diarrheafor the past 3 days. Which of the following is an appropriate action for the nurse to take? A. Offer chicken broth. B. Initiate oral rehydration therapy. C. Start hypertonic IV solution. D. Keep NPO until the diarrhea subsides.

B. CORRECT: Oral rehydration therapy is recommended to replace lost electrolytes for children who have diarrhea.

A nurse is caring for an infant who is postoperative following cleft lip and palate repair. Which of the following actions should the nurse take? A. Remove the packing in the mouth. B. Place the infant in an upright position. C. Offer a pacifier with sucrose. D. Assess the mouth with a tongue blade.

B. CORRECT: Placing the infant in an upright position will facilitate drainage and prevent aspiration.

A nurse is teaching a group of parents about Salmonella. Which of the following information should the nurse include in the teaching? (Select all that apply.) A. Incubation period is nonspecific. B. It is a bacterial infection. C. Bloody diarrhea is common. D. Transmission can be from house pets.E. Antibiotics are used for treatment.

B. CORRECT: Salmonella is classified as a bacterial infection. C. CORRECT: Salmonella manifestations include bloody diarrhea, nausea, vomiting, and abdominal cramping. D. CORRECT: Salmonella can be transmitted to children from household pets such as cats, dogs, hamsters, and turtles.

A nurse is caring for a 10‑year‑old child who has acuteglomerulonephritis. Which of the following findings should the nurse report to the provider? A. Serum BUN 8 mg/dL B. Serum creatinine 1.3 mg/dL C. Blood pressure 100/74 mm Hg D. Urine output 550 mL in 24 hr

B. CORRECT: Serum creatinine 1.3 mg/dL is out of the expected reference range for a 10‑year‑old child, and should be reported to the provider.

A nurse is caring for an infant who has obstructive uropathy. Which of the following findings should the nurse expect? (Select all that apply.) A. Decreased urine flow B. Urinary tract infection C. Metabolic alkalosis D. Concentrated urine E. Hydronephrosis

B. CORRECT: Urinary tract infection is a manifestation of obstructive uropathy. E. CORRECT: Hydronephrosis is a manifestation of obstructive uropathy.

A nurse is caring for a 2 year old child who has a heart defect and is scheduled for cardiac catheterization. Which of the following actions should the nurse take? A. Place on NPO status for 12 hr prior to the procedure B. Check for iodine or shellfish allergies prior to the procedure C. Elevate the affected extremity following the procedure D. limit fluid intake following the procedure

B. Check for iodine or shellfish allergies prior to the procedure

A nurse is teaching a group of parents about complications of communicable diseases. Which of the following communicable diseases may lead to pneumonia? (Select all that apply.) A. Rubella (German measles) B. Rubeola (measles) C. Pertussis (whooping cough) D. Varicella (chickenpox) E. Mumps

B. Complications of rubeola include ear infections, pneumonia, diarrhea, encephalitis,and death. C. Complications of pertussis include: infants and children - pneumonia, convulsions,apnea, encephalopathy, and death; teens and adults - weight loss, loss of bladder control, passing out, and rib fractures. D. Complications of varicella include dehydration, pneumonia, bleeding problems,bacterial infection of the skin, sepsis, toxic shock syndrome, bone or joint infections, and death.

A nurse is assessing an infant who has heart failure. Which of the following findings should the nurse expect? SATA. A. Bradycardia B. Cool extremities C. Peripheral edema D. Increased urinary output E. Nasal flaring

B. Cool extremities C. Peripheral edema E. Nasal flaring

A nurse is caring for a child with enuresis. Which of the following is a complication of enuresis? A. Urinary tract infections B. Emotional problems C. Urosepsis D. Progressive kidney disease

B. Emotional problems are a complication of enuresis.

A nurse is caring for an adolescent client who has mononucleosis. The nurse assesses fever, fatigue, swollen lymph nodes, sore throat, and a sore upper abdomen. Which of the following instructions should the nurse discuss with the adolescent and her parents? (Select all that apply.) A. Take antibiotics until symptoms subside. B. Drink plenty of liquids. C. Avoid participating in strenuous activities. D. Allow for periods of rest. E. Take aspirin as needed for fever and discomfort. F. Gargle with saltwater every 2 to 3 hr.

B. Fluids are encouraged to prevent dehydration with illness. C. The spleen could rupture as a result of injury. Therefore, strenuous activities should be avoided. D. Fatigue is common in clients who have mononucleosis. Therefore, allowing for periods of rest facilitates healing. F. It can soothe discomfort associated with a sore throat.

A nurse is providing teaching to the parent of a child who has attention-deficit/hyperactivity disorder.The nurse should include which of the following as a characteristic of impulsivity? A. Loses things B. Frequently interrupts C. Is easily distracted D. Talks excessively

B. Frequently interrupting is a characteristic of impulsivity.

An infant who has clinical manifestations of acute otitis media (AOM) is brought to an outpatient facility by his parent. The nurse should recognize that which of the following factors places the infant at risk for otitis media? (Select all that apply.) A. Breastfeeding without formula supplementation. B. Attends day care 4 days per week. C. Immunizations are up to date. D. History of a cleft palate repair. E. Parents smoke cigarettes outside.

B. Infants who attend day care have an increased risk of OM because of the exposure to multiple people. D. Infants born with cleft palate are more prone to AOM because micro-organisms can easily enter the eustachian tubes. E. Exposure to secondhand smoke increases an infant's risk for AOM.

A nurse is assessing an infant. Which of the following findings are clinical manifestations of acute otitis media? (Select all that apply.) A. Decreased pain in the supine position B. Rolling head side to side C. Loss of appetite D. Increased sensitivity to sound E. Crying

B. Infants who have acute otitis media will roll their head side to side because of the pain and pressure in the ear. C. Infants who have acute otits media will exhibit a loss of appetite due to the pain and pressure in the ear. E. Infants who have acute otitis media will exhibit crying and irritability from the pain.

A nurse is caring for a child who has had watery diarrhea for the past 3 days. Which of the following is the most appropriate action for the nurse to take? A. Offer high fiber foods. B. Initiate oral rehydration therapy. C. Start hypertonic IV solution. D. Keep NPO until the diarrhea subsides.

B. Initiate oral rehydration therapy.

A nurse is assessing an infant who has a suspected urinary tract infection. Which of the following are anticipated findings? (Select all that apply.) A. Increase in hunger B. Irritability C. Decrease in urination D. Vomiting E. Fever

B. Irritability is a clinical manifestation in an infant with a urinary tract infection. D. Vomiting is a clinical manifestation in an infant with a urinary tract infection. E. Fever is a clinical manifestation in an infant with a urinary tract infection.

A nurse is caring for a child who has Hirschsprung's disease. Which of the following actions should the nurse take? A. Encourage a high-fiber, low-protein, low-calorie diet. B. Prepare the family for surgery. C. Place an NG tube for decompression. D. Initiate bed rest

B. Prepare the family for surgery.

A nurse is caring for a toddler who has rhinitis, cough, and diarrhea for 2 days. Upon assessment, it is noted that the tympanic membrane has a orange discoloration and decreased movement. Which of the following is an appropriate statement for the nurse to make? A. "Your child has an ear infection that requires antibiotics." B. "Your child could experience transient hearing loss." C. "Your child will need to be on a decongestant until this clears." D. "Your child will need to have a myringotomy."

B. Rhinitis, cough, diarrhea, and an orange discoloration of the tympanic membrane are clinical findings of OME. Transient hearing loss is a complication of OME.

When conducting a physical examination of a child with suspected Kawasaki disease, which finding would the nurse expect to assess? A. Hirsutism or striae B. Strawberry tongue C. Malar rash D. Café au lait spots

B. Strawberry tongue

A nurse is assessing a child who has a urinary tract infection. Which of the following are clinical manifestations of a urinary tract infection? (Select all that apply.) A. Night sweats B. Swelling of the face C. Pallor D. Pale colored urine E. Fatigue

B. Swelling of the face is a clinical manifestation in children with a urinary tract infection. C. Pallor is a clinical manifestation in children with a urinary tract infection. E. Fatigue is a clinical manifestation in children with a urinary tract infection.

A 3 year old is diagnosed with autism spectrum disorder. What information reported by the parents would support the diagnosis? A. The child only liked to play with his siblings. B. The child did not like to be cuddled or picked up as an infant. C. The child likes to look at books as they are read to him. D .The child will only play at the childcare center after the parents leave.

B. The child did not like to be cuddled or picked up as an infant.

The parents of a child diagnosed with ADHD are very upset. They state "we have never thought of him as having a low IQ level." What should the nurse explain to reduce their knowledge deficit? A. It is difficult to have identified this deficit in their own child. B. The majority of children with ADHR do not have a low intelligence level. C. There are counselors to help with their child's lack of intelligence. D. The child will be able to participate in special education classes due to his intelligence level.

B. The majority of children with ADHR do not have a low intelligence level.

A nurse is teaching a parent of a child who has a urinary tract infection. Which of the following should the nurse include in the teaching? (Select all that apply.) A. Wear nylon underpants. B. Avoid bubble baths. C. Empty bladder completely with each void. D. Provide information about clinical manifestations of infection. E. Wipe perineal area back to front.

B. The nurse should discuss avoiding bubble baths. C. The nurse should discuss the need to completely empty the bladder with each void. D. The nurse should review the clinical manifestations of infection.

A nurse is caring for a child who has depression. Which of the following findings are associated with this diagnosis? (Select all that apply.) A. Prefers being with peers B. Weight loss C. Report of low self-esteem D. Sleeping more than usual E. Hyperactivity

B. Weight loss or gain are findings associated with depression. C. Low self-esteem is a finding associated with depression. D. Sleeping more than usual is a finding associated with depression.

A nurse is reviewing the maternal history of a large-for-gestational-age (LGA) newborn. Which factor, if noted in the maternal history, would the nurse identify as possibly contributing to the birth of this newborn? A. substance use disorder B. diabetes C. preeclampsia D. infection

B. diabetes

What would be appropriate for the nurse to document in a child suffering from meconium aspiration syndrome? A. heart rate as normal B. respirations as increased and high C. skin as pink D. chest expansion as normal

B. respirations as increased and high

An infant is prescribed digoxin. What should the nurse explain to the parents regarding the action of this medication? A. increases the heart rate B. slows and strengthens the heartbeat C. thickens the walls of the myocardium D. prevents subacute bacterial endocarditis

B. slows and strengthens the heartbeat

Which response by a child suggests that he understands an important fact regarding the administration of a prescribed antibiotic for impetigo? A."I have to take this until the itchiness stops." B."I'll continue the drug until it's all gone." C."I'll have to take the drug at least 4 days." D."Taking the drug will prevent itchiness."

B."I'll continue the drug until it's all gone."

A nurse is caring for a client who has a superficial partial-thickness burn. Which of the following actions should the nurse take? A.Administer IV infusion of 0.9% sodium chloride. B.Apply cool, wet compresses to the affected area. C.Clean the affected area using a soft-bristle brush. D.Administer morphine sulfate.

B.Applycoolwetcompresses

The nurse notices that Sage was seen in the emergency department last year for acetaminophen poisoning. The hospital's algorithm for dealing with acetaminophen overdose should specify what action? A.Advise the parents their child must never take acetaminophen again. B.Be prepared to administer either acetylcysteine or activated charcoal. C.Palpate the child's abdomen to assess whether the bladder feels tender. D.Administer potassium chloride elixir immediately.

B.Be prepared to administer either acetylcysteine or activated charcoal.

A nurse has been assigned to the care of two newborn infants who are 38 weeks' gestation. One is small-for-gestational-age and one is large-for gestational-age. The nurse knows that they both are most likely at risk for all the following complications except A.Polycythemia B.Intraventricular hemorrhage C.Hypoglycemia D.Hyperbilirubinemia

B.Intraventricular hemorrhage

Parents are taught to place their infants on their backs to sleep to prevent sudden infant death syndrome (SIDS). Which disorder would require the infant to be placed on the side to sleep to avoid airway obstruction? A.Thyroglossal cyst B.Pierre Robin syndrome C.Preoperative cleft lip D.Repaired cleft lip

B.Pierre Robin syndrome

A nurse is providing teaching to the caregiver of an infant who has a prescription for digoxin. Which of the following instructions should the nurse include? A. "Do not offer your baby fluids after giving the medication." B. "Digoxin increases your baby's heart rate." C. "Give the correct dose of medication at regularly scheduled times." D. "If your baby vomits a dose, you should repeat the dose to ensure that the correct amount is received."

C. "Give the correct dose of medication at regularly scheduled times."

A nurse is caring for a client who has a moderate burn. Which of the following is an appropriate action for the nurse to take? A. Maintain immobilization of the affected area. B. Expose affected area to the air. C. Initiate a high-protein, high-calorie diet. D. Implement contact isolation.

C. A high-protein, high-calorie diet is initiated to meet increased metabolic demands and promote healing.

On assessment, which finding would the nurse expect in a child with allergic rhinitis? A. Reddened, tender nasal mucosa B. Erythematous lesions on palate C. Blackened circles under eyes D. A thick, purulent nasal discharge

C. Blackened circles under eyes

A nurse is teaching the parent of a newborn how to treat the newborn's plagiocephaly. Which of the following statements by the parent indicates an understanding of the teaching? A. "I should put my baby to sleep on her belly duringher afternoon nap." B. "I should ensure my baby's head is in the same positioneach time she sleeps." C. "I should have my baby wear the prescribed helmet 23 hours a day." D. "I should allow my baby to sleep in her infant swing."

C. CORRECT: A newborn who has plagiocephalyshould wear the prescribed helmet 23 hr/day.

A nurse is teaching a child who has type 1 diabetes mellitus about self care. Which of the following statements by the child indicates understanding of the teaching? A. "I should skip breakfast when I am not hungry." B. "I should increase my insulin with exercise." C. "I should drink a glass of milk when I am feeling irritable." D. "I should draw up the NPH insulin into the syringe before the regular insulin."

C. CORRECT: An early manifestation of hypoglycemia is irritability. Drinking a glass of milk, which is approximately 15 g of carbohydrates, indicates understanding of the teaching.

A nurse is caring for a child who has type 1 diabetes. Which of the following is a clinical manifestation of diabetic ketoacidosis? (Select all that apply.) A. Blood glucose 58 mg/dL B. Weight gain C. Dehydration D. Mental confusion E. Fruity breath

C. CORRECT: Clients who have diabetic ketoacidosis experience osmotic diuresis because of the electrolyte shift. D. CORRECT: Clients who have diabetic ketoacidosis experience mental confusion because of the electrolyte shift. E. CORRECT: Clients who have diabetic ketoacidosis experience fruity breath because of the body's attempt to eliminate ketones.

A nurse is admitting a child who has HIV. The nurse should identify which of the following findings as an indication that the child is in the mildly symptomatic category of HIV? (Select all that apply.) A. Herpes zoster B. Anemia C. Dermatitis D. Hepatomegaly E. Lymphadenopathy

C. CORRECT: Dermatitis is a manifestation of a child who is mildly symptomatic. D. CORRECT: Hepatomegaly is a manifestation of a child who is mildly symptomatic. E. CORRECT: Lymphadenopathy is a manifestation of a child who is mildly symptomatic.

A parent of a school-age child with GH deficiency asks the nurse how long his son will need to take injections for his growth delay. Which of the following is an appropriate response by the nurse? A. "Injections are usually continued until age 10 for girls and age 12 for boys." B. "Injections continue until your child reaches the fifth percentile on the growth chart." C. "Injections should be continued until there is evidence of epiphyseal closure." D. "The injections will need to be administered throughout your child's entire life."

C. CORRECT: Injections are continued until there is evidence of epiphyseal closure on radiographic tests.

A nurse is providing teahcing to the caregiver of an infant who has a prescription for digoxin. Which of the following instructions should the nurse include? A. Do not offer your baby fluids after giving the medication B. Digoxin increases your baby's heart rate C. Give the correct dose of medication at regularly scheduled times. D. If your baby vomits a dose, you should repeat the dose to ensure that the correct amount is received.

C. Give the correct dose of medication at regularly scheduled times.

A nurse is caring for a 2-year-old child who has had three ear infections in the past 5 months. The nurse should know that the child is at risk for developing which of the following as a long-term complication? A. Balance difficulties B. Prolonged hearing loss C. Speech delays D. Mastoiditis

C. Speech delay is a common complication of otitis media.

A nurse is caring for a child who is experiencing respiratory distress. Which of the following findings are early manifestations of respiratory distress? (Select all that apply.) A. Bradypnea B. Peripheral cyanosis C. Tachycardia D. Diaphoresis E. Restlessness

C. Tachycardia D. Diaphoresis E. Restlessness A and B are signs of late respiratory distress.

A 16 year old female has been diagnosed with bulimia. Upon physical exam, which finding would support this diagnosis? A. Weight below the fifth percentile B. Hair loss C. Tooth enamel erosion D, RUQ pain

C. Tooth enamel erosion

A nurse is assessing a young child and suspects coarctation of the aorta based on which finding? A. Excessive crying B. Diastolic murmur C. Unequal upper and lower extremity pulses D. Hypotension

C. Unequal upper and lower extremity pulses

A nurse is preparing to administer varicella vaccine to an adolescent. Which of the following questions should the nurse ask to determine whether there is a contraindication to administering the vaccine? A. "Do you have an allergy to eggs?" B. "Have you ever had encephalopathy following immunizations?" C. "Are you currently taking corticosteroid medication?" D. "Do you have a hypersensitivity to yeast?"

C. Varicella vaccine is contraindicated in clients who are currently taking corticosteroid medications.

A nurse is assessing a child who has chronic renal failure. Which of the following findings should the nurse expect? A. Flushed face B. Hyperactivity C. Weight loss D. Delayed growth

C. Weight loss

A newborn was resuscitated at birth due to poor respiratory effort. Which assessment data would concern the nurse the most at 6 hours after birth? A.Lethargy at first feeding attempt B.Cyanotic hands and feet C.Body temperature of 95.0°F D.Respirations of 40 breaths/min

C.Body temperature of 95.0°F this could be wrong

The BEST way in which a nurse can detect congenital hydrocephalus prior to brain tissue damage is to A. Assess for brow bulging. B. Perform a transillumination of the infant's skull. C.Measure the infant's head circumference at birth and then compare the measurement prior to discharge. D. Assess fontanelles for width and tension.

C.Measure the infant's head circumference at birth and then compare the measurement prior to discharge.

A child is diagnosed with erythema infectiosum. Which report by the mother assisted with the diagnosing of this illness? A.High fever for 2 days and blister in mouth B.Red palms and soles of feet C.Red cheeks and rash D.Joint stiffness and reddened sclera

C.Red cheeks and rash

T Lymphocytes

Cell mediated immunity

Allergies: Hypersensitivity TYPE 4

Cell-Mediated Hypersensitivity Delayed. Contact dermatitis, transplant graft reaction

Respiratory Arrest

Complete cessation of respiration

Airway obstruction

Could be due to aspiration, foreign body ingestion, or anaphylactic reaction

Allergies: Hypersensitivity TYPE 2

Cytotoxic Response Antigen destruction Hemolytic anemia, transfusion reaction, erythroblastosis fetalis

A nurse is preparing to administer immunizations to a 6-month-old infant. Which of the following is an appropriate action for the nurse to take in providing atraumatic care? A. Administer 81 mg of aspirin. B. Use the Z-track method when injecting. C. Ask the parents to leave the room during the injection. D. Provide sucrose solution on the pacifier.

D. Allowing an infant to suck on a pacifier with sucrose solution can decrease pain with immunizations and is an appropriate action for the nurse to take in providing atraumatic care.

A school-age child has a deficiency in humoral immunity. Which level would the nurse expect to be reduced? A. Lymphokines B. Haptens C. Allergens D. Antibodies

D. Antibodies

A nurse is reviewing the medical record of a newborn who has necrotizing enterocolitis (NEC). Which of the following findings is a risk factor for NEC? A. Macrosomia B. Transient tachypnea of the newborn (TTN) C. Maternal gestational hypertension D. Gestational age 36 weeks

D. CORRECT: A gestational age of 36 weeks, or apreterm birth, places a newborn at risk for NEC.

A nurse is developing a plan of care for a newborn who hashyperbilirubinemia and is to undergo phototherapy. Which of the following actions should the nurse include in the plan of care? A. Reposition the newborn every 4 hr. B. Lotion the newborn's skin twice per day. C. Check the newborn's temperature every 8 hr. D. Remove the newborn's eye mask during feedings.

D. CORRECT: A newborn undergoing phototherapy should have the eye mask removed for each feeding to allow for bonding and assessment of the newborn's eyes.

A nurse is caring for a child who has AIDS. Which of the following isolation precautions should the nurse implement? A. Contact B. Airborne C. Droplet D. Standard

D. CORRECT: Standard isolation precautions are used to protect transmission of disease that is bloodborne or present in a body fluid.

A nurse is assessing a child who has chronic renal failure.Which of the following findings should the nurse expect? A. Flushed face B. Hyperactivity C. Weight gain D. Delayed growth

D. CORRECT: The nurse should expect the child to exhibit delayed growth.

A nurse is teaching a school‑age child who has diabetes mellitus about insulin administration. Which of the following should the nurse include in the teaching? A. "You should inject the needle at a 30‑degree angle." B. "You should combine your glargine and regular insulin in the same syringe." C. "You should aspirate for blood before injecting the insulin." D. "You should give four or five injections in one area before switching sites."

D. CORRECT: The nurse should instruct the client to administer four or five injections about 2.5 cm (1 in) apart before switching to another site.

A nurse is planning care of a child who has a urinary tract infection. Which of the following interventions should the nurse include? A. Administer an antidiuretic. B. Restrict fluids. C. Evaluate the child's self-esteem. D. Encourage frequent voiding.

D. Encourage frequent voiding.

A 7-year-old is diagnosed with type 1 diabetes mellitus. Of the many labs drawn at the time of diagnosis, which test is specific to this type of diabetes? A. Arterial blood gases B. BUN C. Ammonia level D. Glycosylated hemoglobin

D. Glycosylated hemoglobin also called AIC

A nurse is teaching a group of parents about infants who have failure to thrive. Which of the following characteristics should be included in the teaching? A. They have been neglected. B. They come from an impoverished environment. C. They manifest colicky behaviors. D. They exhibit developmental delays.

D. Infants who have failure to thrive exhibit developmental delays as a result of decreased nutritional intake needed for brain development.

A nurse is planning care of a child who has a urinary tract infection. Which of the following should the nurse include? A. Administer antidiuretic. B. Restrict fluids. C. Evaluate the child's self-esteem. D. Encourage frequent voiding.

D. It's important to encourage frequent voiding. This assists in flushing the bacteria through the urinary system.

A nurse is teaching a group of parents about communicable diseases. Which of the following is the most appropriate method to prevent a communicable disease? A. Handwashing B. Avoiding persons with active disease C. Covering your cough D. Obtaining immunizations

D. Obtaining immunizations has decreased the rate of communicable diseases and is the best method to prevent further spread of illness.

A nurse is caring for a toddler who has acute otitis media. Which of the following is the priority action for the nurse to take? A. Provide emotional support to the family. B. Educate the family on care of the child. C. Prevent clinical complications. D. Administer analgesics.

D. The priority action the nurse should take when using Maslow's hierarchy of needs is to meet the toddler's physiological need first. Therefore, administering analgesics to alleviate or decrease physical pain is the priority action for the nurse to take.

In the delivery of an infant with meconium-stained fluid, the nurse should anticipate which measure being taken first? A.Bag and mask ventilations B.Immediate intubation after delivery C.Oxygen applied by mask D.Bulb syringe used to suction mouth and nose before delivery of shoulders

D.Bulb syringe used to suction mouth and nose before delivery of shoulders

Maia tells the nurse her baby was born on an unlucky day because the baby born before hers had his bowel outside his body. What is the most important nursing consideration at birth in the care of a baby with gastroschisis? A.Position the infant on his or her stomach so the intestine is well contained. B.Wrap the intestines with chilled gauze to prevent intestinal swelling. C.Keep the infant seated upright under a radiant warmer for warmth. D.Contain the intestine in a warmed, sterile saline-lined bowel bag.

D.Contain the intestine in a warmed, sterile saline-lined bowel bag. could be wrong

Atopic Dermatitis

Excess inflammation; dry skin, redness, and itching from allergies and irritants. Eczema

Gold Standard test for Head Injury

Head Circumference

B-Lymphocyte Deficiencies

Humoral defects are generally not seen until 4-6 months of age because of the presence of maternal antibodies which cross the placenta. -b cells are incapable of maturing into antibodies -treated w/ monthly infections of IVIG

Allergies: Hypersensitivity TYPE 3

Immune Complex Inflammatory response Rheumatoid arthritis, systemic lupus

Respiratory Failure

Inability to maintain adequate oxygenation in the blood

T-Lymphocyte Deficiencies

Inadequate numbers/functioning of one or more types of T lymphocytes can result in congenital heart disease, aplastic or hypoplastic thymus, hypokalemia, and cleft lip and/or palate treatment: IVIG infusion

Hemolytic Disease (hyperbilirubinemia)

Increased bilirubin levels causing jaundice. -treatment: phototherapy, I&Os, encourage feeding, breastfeeding.

Respiratory Insufficiency/Distress

Increased wob, but mostly adequate gas exchange

Severe Combined T and B Lymphocyte Deficiency

Inherited rare disorder with defects in both B and T cells treatment: IVIG treatment until stem cell transplant

Cushing Syndrome

Overproduction of cortisol from the adrenal gland -increased glucose production

IgE immunoglobulin

Produced by plasma cells in skin, tonsil, respiratory, and digestive systems. Overproduction is responsible for allergic reactions

Food Refusal or Aversion

Significant failure to gain weight or weight loss, without medical reason or lack of food

Congestive Heart Failure (CHF)

The inability of the heart to supply adequate oxygenated blood to meet the metabolic demands of the body. -can be one side or both

Pierre Robin Sequence/Syndrome

Triad Syndrome: 1. micrognathia (small mandible) 2. cleft palate 3. Glossoptosis (tongue malposition downward) -surgical repair

HIV/AIDS

Virus HIV-1 and HIV-2. Attacks CD4 Helper T cells. -HIV antigen test/western blot for diagnosis (gold standard) -treated with antiretrovial cocktail

Full Thickness Burn

a burn in which all the layers of the skin are damaged. There are usually areas that are charred black or areas that are dry and white. Also called a third-degree burn.

Partial Thickness Burn

a burn involving the epidermis and dermis that usually involves blisters; commonly called a second-degree burn

Superficial Burn

a burn that involves only the epidermis, the outer layer of the skin. It is characterized by reddening of the skin and perhaps some swelling. An example is a sunburn. Also called a first-degree burn.

Encopresis

a childhood disorder characterized by repeated defecating in inappropriate places, such as one's clothing -treatment: history of usual bowel evacuation habits, number of bowel accidents and times they occcur; rectal exam for propr anal sphincter control

Tetralogy of Fallot

a congenital malformation of the heart involving four distinct defects: 1. pulmonary stenosis 2. ventricular septal defect 3. overriding aorta 4. Right ventricular hypertrophy -decreased pulmonary blood flow -cyanotic -systolic murmur at the left upper sternal border, episodes of acute cyanosis and hypoxia -*hypercyanotic spells? --> knee to chest position, supplemental oxygen, immediate ER intervention if no improvement

Autism Spectrum Disorder

a disorder that appears in childhood and is marked by significant deficiencies in communication and social interaction, and by rigidly fixated interests and repetitive behaviors

Phenylketonuria

a genetic disorder in which the essential digestive enzyme phenylalanine hydroxylase is missing. affects the building blocks of protein

Atrial Septal Defect

a hole in the atrial septum that causes blood in the RA and LA to mix. Blood flows from the high-pressure left atrium to the low-pressure right atrium and continues out to the lungs. -increased pulmonary blood flow -may go unnoticed. The child will present with symptoms of pulmonary over circulation (rales, congestion, tiring with activity, poor weight gain, right sided heart enlargement) -Treatment: medical, surgical management, NG feeds

Ventricular Septal Defect (VSD)

a hole in the ventricular septum that causes blood to mix between the RV and LV -increased pulmonary blood flow -loud, harsh holosystic murmur noted at the LLSB -cyanotic -treatment: small defect may close on its own. Diuretics (furosemide), NG feeds, surgical closure, increase caloric density of formula or breastmilk

Attention Deficit Hyperactivity Disorder (ADHD)

a psychological disorder marked by the appearance by age 7 of one or more of three key symptoms: extreme inattention, hyperactivity, and impulsivity

A 13-year-old adolescent with hyperthyroidism who takes antithyroid medication has a sore throat and a fever. The parent calls the nurse and asks what to do. Which is the best response from the nurse? a. "Please take your child straight to the emergency department." b. "Fever and sore throat may be side effects of the medication." c. "Give your child ibuprofen according to the instructions on the box." d. "Offer your child at least 8 ounces of clear fluids and call back tomorrow."

a. "Please take your child straight to the emergency department."

A nurse is caring for a 7-year-old girl scheduled for an intravenous pyelogram (IVP). Which action would be the priority before the test? a. Checking with the parents for any allergies b. Ensuring adequate hydration c. Giving the girl an enema d. Screening her for pregnancy

a. Checking with the parents for any allergies

The nurse is caring for a 14-year-old girl with atrial fibrillation. Which medication would the nurse expect to be prescribed? a. Digoxin b. Alprostadil c. Furosemide d. Indomethacin

a. Digoxin

When evaluating the hemogram of an 8-month-old infant, the nurse would identify which type of hemoglobin as being the predominant type? a. Hemoglobin A b. Hemoglobin F c. Hemoglobin A2 d. Hemoglobin S

a. Hemoglobin A

A nurse prepares a menu for a client with Crohn disease. What is the focus of dietary management for this disease? a. High carbohydrate, high protein b. Low calorie, high carbohydrate c. High calorie, high fiber d. Low fiber, low calorie

a. High carbohydrate, high protein

A 4-month-old infant is found to be anemic. Which is the most likely cause of anemia in this child? a. Inadequate intake of dietary iron in the mother during late gestation b. Inadequate intake of dietary iron in the infant c. Sickle cell anemia d. Thalassemia

a. Inadequate intake of dietary iron in the mother during late gestation

A school-aged child is brought into the emergency room, and the preliminary diagnosis is acute adrenocortical insufficiency. Which of the interventions below should the nurse implement first? a. Insert an IV line in preparation for giving IV fluids and cortisol. b. Prepare the child for admission to the pediatric intensive care unit. c. Arrange for a bedside electrocardiogram to be performed. d. Administer oxygen via a nonrebreather mask.

a. Insert an IV line in preparation for giving IV fluids and cortisol.

A 20-year-old client gave birth to a baby boy at 43 weeks' gestation. What might the nurse observe in the newborn during routine assessment? a. The newborn may look wrinkled and old at birth. b. The infant may have excess of lanugo and vernix caseosa. c. The testes in the child may be undescended. d. The newborn may have short nails and hair.

a. The newborn may look wrinkled and old at birth.

While assessing a child with end-stage kidney disease, the nurse notes that the child has fallen into a coma. The nurse interprets this finding as resulting from which complication? a. Uremia b. Metabolic acidosis c. Immunosuppression d. Hypocalcemia

a. Uremia

In caring for a child with nephrotic syndrome, which intervention will be included in the child's plan of care? a. Weighing on the same scale each day b. Ambulating 3 to 4 times a day c. Increasing fluid intake by 50 ml per hour d. Testing the urine for glucose levels regularly

a. Weighing on the same scale each day

The nurse is working in the newborn nursery and receives a report from the delivery room nurse. When receiving the hand-off report, which situation(s) will the nurse identify as common in the assessment findings of developmental dysplasia of the hip (DDH)? Select all that apply. a. breech position at or after 34 weeks' gestation b. transverse lie c. female gender d. male gender e. oligohydramnios f. family history of DDH

a. breech position at or after 34 weeks' gestation c. female gender e. oligohydramnios f. family history of DDH

Nurses will need to teach the parents of the infant being treated for Hirschsprung disease (aganglionic megacolon) to: a. care for a temporary colostomy. b. thicken formula feedings. c. avoid use of a pacifier. d. carefully monitor heart rate.

a. care for a temporary colostomy.

A nurse is teaching a parent about post traumatic stress disorder *PTSD. Which of the following information should the nurse include in the teaching? select all that apply a. children who have PTSD can benefit from psychotherapy b. a manifestation of PTSD is phobias c. Personality disorders are a complication of PTSD d. PTSD develops following a traumatic event E. there are 6 stages of PTSD

a. children who have PTSD can benefit from psychotherapy b. a manifestation of PTSD is phobias d. PTSD develops following a traumatic event

Insulin deficiency, in association with increased levels of counter-regulatory hormones and dehydration, is the primary cause of: a. diabetic ketoacidosis. b. ketone bodies. c. ketonuria. d. glucosuria.

a. diabetic ketoacidosis

A client has given birth to a small-for-gestational-age (SGA) newborn. Which finding would the nurse expect to assess? a. head larger than body b. round flushed face c. brown lanugo body hair d. protuberant abdomen

a. head larger than body

A nurse is instructing parents of a child with allergies the events of a type I hypersensitivity reaction. Which would the nurse describe as being responsible for vessel congestion and edema? Select all that apply. a. histamine b. leukotrienes c. slow-reacting substance of anaphylaxis (SRS-A) d. chemotactic substances e. IgG immunoglobulin

a. histamine b. leukotrienes

Which laboratory test result would be most important for the nurse to assess in a child who is suspected of having a urinary tract infection? a.urinalysis b. chemical reagent strip testing c. urine specific gravity level d. serum blood urea nitrogen (BUN) level

a.urinalysis

Contractility

ability to modulate the rate and contractions of the heart. The force in which it's contractions occur.

Retinopathy of Prematurity

acquired ocular disease that leads to partial or total blindness in children. vasoconstriction of retinal blood cells that are immature due to hypoxia

Kawasaki Disease

acute febrile syndrome associated with generalized vasculitis (inflammation of blood vessels) affecting all blood vessels throughout the body, including coronary arteries. -prolonged fever of 5 or more days, -four or more of the following symptoms: erythema, polymorphous exanthema, bilateral conjunctivitis w/o exudate, changes in lips and mouth, STRAWBERRY TOUNGE, cervical lymphadenopathy -treatment: IVIG, high dose aspirin

Myocarditis

acute or inflammatory process of the myocardium (heart muscle) -chest pain, exertional dyspnea, fatigue, palpitation in assoc. w/ acute CHR

Herpes Zoster (Shingles)

an acute viral infection characterized by painful skin eruptions that follow the underlying route of an -inflamed nerve. Shingles. -Viral Infection -Airborne Precautions

Allergic Rhinitis

an allergic reaction to airborne allergens that causes an increased flow of mucus seasonal patterns (called perennial allergic rhinitis) -dark shiners (dark circles under the eyes caused by congestion) -treatment: avoidance, antihistamines, leukotriene inhibitors, corticosteroids

Allergies: Hypersensitivity TYPE 1

anaphylaxis (anaphylactic shock) immediate allergies, asthma, atopic dermatitis, anaphylaxis -treatment: epinephrine, bronchodilator (albuterol), corticosteroids sometimes as second line

Drowning

asphyxiated while submerged in at least one inch of water

Meconium Aspiration Syndrome

aspiration of the meconium during birth -treatment: suction, oxygen, positive pressure ventilation, antibiotics, surfactant

IgA immunoglobulin

assist in tears, secretions, and colostrum for newborns

Plagiocephaly

asymmetry of the skull

hyperthyroidism (Graves Disease)

autoimmune. The thyroid is stimulated to overproduce T4 (thyroid hormone) -nervousness, tremors, loss of muscle strenth, HTN, constant hunger without weight gain, exophthalmos (eyes popping out) -treatment: PTU

A nurse is preparing a presentation on preventing toxic shock syndrome for a health class of high school students. Which information will the nurse include in the presentation? Select all that apply. a. Use tampons made of synthetic materials. b. Change tampons at least every 4 hours. c. Insert only one tampon at a time. d. Avoid using feminine hygiene sprays. e. Try using deodorant tampons instead of unscented ones. f. Use a superabsorbent tampon; they can remain in place for longer.

b. Change tampons at least every 4 hours. c. Insert only one tampon at a time. d. Avoid using feminine hygiene sprays.

While assessing a neonate with a ventricular septal defect (VSD), the nurse notes crackles and retractions. The nurse obtains the following vital signs: temp 100.2°F (38°C), pulse 134 bpm, respirations 64 breaths/minute, oxygen saturation 97% on room air. What will the nurse do first? a. Advise the mother to bottle feed. b. Give furosemide intravenously. c. Administer acetaminophen rectally. d. Apply oxygen 10 liters/min (LPM) via oxyhood.

b. Give furosemide intravenously.

The nurse is assessing a newborn and suspects developmental dysplasia of the hip (DDH). For which sign is the nurse prioritizing in this potential diagnosis? a. Symmetry of the gluteal skin folds b. Limited abduction of the affected hip c. Lengthening of the femur d. Bilateral adduction of the legs

b. Limited abduction of the affected hip

The location of the kidneys in the child in relationship to the location of the kidneys in the adult makes which fact a greater likelihood in the child? a. The adult has less fat to cushion the kidney. b. The child has a greater risk for trauma to the kidney. c. The child has more frequent urges to empty the bladder. d. The adult has a greater chance of retaining fluids than the child.

b. The child has a greater risk for trauma to the kidney.

A parent brings the 16-year-old adolescent into the clinic with a temperature of 101.6°F (38.6°C) and abdominal pain. The nurse suspects that the adolescent has pelvic inflammatory disease. Which assessment question(s) will the nurse ask to confirm or deny the suspicion? Select all that apply. a. Do you use superabsorbent tampons during your menstrual periods? b. What is the type or quality of pain in your abdomen? c. When was your last menstrual period? d. Have you been voiding clear yellow urine? e. Are you having any unusual vaginal discharge?

b. What is the type or quality of pain in your abdomen? c. When was your last menstrual period? e. Are you having any unusual vaginal discharge?

A 3-year-old child is exhibiting irritability, fever, and decreased appetite. A recent history of which of the following would make the nurse suspicious of a urinary tract infection (UTI)? a. lymphadenopathy b. abdominal pain c. rash d. leg pain

b. abdominal pain

A nurse is providing instruction to the teacher of a child who has attention deficit/ hyperactivity disorder *ADHD. Which of the following classroom strategies should the nurse include in the teaching? select all that apply a. eliminate testing b. allow for regular breaks c. combine verbal instruction with visual cues d. establish consistent classroom rules e. increase stimuli in the environment

b. allow for regular breaks c. combine verbal instruction with visual cues d. establish consistent classroom rules

A young client with head trauma arrives at the emergency department. The injury is severe, and the child has rapid brain compression. Bleeding is arterial. The nurse recognizes that this client has which type of head injury? a. subdural hematoma b. epidural hematoma c. skull fracture d. concussion

b. epidural hematoma

The immune system works to destroy pathogens by helping the body get rid of or resist the invasion of foreign materials The blood cells that surround, ingest, and neutralize the pathogens are: a. erythrocytes. b. macrophages. c. platelets. d. lymphocytes.

b. macrophages.

A nurse is caring for a child who is experiencing heart failure. Which assessment data was most likely seen when initially examined? a. bradycardia b. tachycardia c. splenomegaly d. polyuria

b. tachycardia

A nurse is caring for a child who has depression. Which of the following findings should the nurse expect? select all that applies a. preferring being with peers b. weight loss or gain c. report of low self esteem d. sleeping more than usual e. hyperactivity

b. weight loss or gain c. report of low self esteem d. sleeping more than usual

Nursing Intervention for choking for infants

back blows and chest thrusts

left sided heart failure

blood is coming in from the lungs, so these symptoms represent too much fluid in the lungs. -accessory muscles, retractions, rales

Late preterm infant

born between 34-37 weeks gestation

Early preterm infant

born prior to 34 weeks and after 24 weeks

The nurse is providing education to the parents of a child diagnosed with pinworms. Which statement is most important for the nurse to include in the teaching? a. "Seal the child's clothing in a plastic bag for at least 10 days." b. "Be sure your child wears shoes at all times." c. "Make sure your child washes hands before eating." d. "After applying this special cream, leave it on for about 8 to 10 hours."

c. "Make sure your child washes hands before eating."

The nurse is caring for a comatose school-age child receiving gastrostomy tube feedings. The nurse aspirates 15 ml of stomach contents prior to administering a feeding. What is the appropriate action by the nurse? a. Discard the stomach contents and continue with the feedings as prescribed. b. Replace the stomach contents and hold the feeding. c. Replace the stomach contents and continue with the feedings as prescribed. d. Discard the stomach contents and notify the health care provider of the aspiration amount.

c. Replace the stomach contents and continue with the feedings as prescribed.

The nurse observes a neonate born at 28 weeks' gestation. Which finding would the nurse expect to see? a. The skin is pale, and no vessels show through it. b. Creases appear on the interior two-thirds of the sole. c. The pinna of the ear is soft and flat and stays folded. d. The neonate has 7 to 10 mm of breast tissue.

c. The pinna of the ear is soft and flat and stays folded.

Which congenital condition is an immediate emergency requiring notification of the health care provider? a. Hypospadias b. Cleft palate c. Tracheoesophageal fistula d. Atrial septal defect

c. Tracheoesophageal fistula

The nurse is caring for a child with rheumatic fever who has polyarthritis. Which lab result would the nurse most anticipate with this child's diagnosis and symptoms? a. increased clotting time b. decreased white blood cell count (WBC) c. increased erythrocyte sedimentation rate (ESR) d. decreased leukocyte count

c. increased erythrocyte sedimentation rate (ESR)

right sided heart failure

causes swelling in extremities

Apnea

cessation of respirations >20 seconds (sometimes w bradycardia or cyanosis)

periventricular/intraventricular hemorrhage

complication of preterm -bleeding in (intra) or around (peri) ventricles

anemia of prematurity

complication of preterm infant -nonmature blood cell producing ability. -treatment: delay cord clamping, minimize blood draws, transfusion

Pica

compulsive eating of nonnutritive substances such as clay or ice. often presents as pice-induced complicaiton -often associated w/ iron deficiency

Congenital Hypothyroidism

condition present at birth that results in lack of thyroid hormones -excessive sleep, enlarged tongue, poor suckling, prolonged jaundice, hypotonic muscles, chronic constipation -low serum T3 and T4; elevated thyroid stimulating factor -treatment: synthetic thyroid hormone

Gastroschisis

congenital fissure of the abdominal wall not at the umbilicus -treatment: surgery within 24 hours of birth, antibiotics, tpn, central line

cell mediated immunity (t lymphocytes)

cytotoxic T cells (killer t cells) bind to the surface of an antigen and destroy it's cell membrane

The nurse is developing a plan of care for an infant with heart failure who is receiving digoxin. The nurse would hold the dose of digoxin and notify the physician if the infant's apical pulse rate was: a. 140 beats per minute. b. 120 beats per minute. c. 100 beats per minute. d. 80 beats per minute.

d. 80 beats per minute.

A mother asks when a preterm infant receives basic immunizations. Which response by the nurse is most accurate? a. Basic immunizations are given according to the chronologic age of an infant. b. The infant will receive basic immunizations before discharge from the NICU. c. The infant's immunizations will all be delayed until 6 months corrected age. d. Basic immunizations are given according to the infant's corrected gestational age.

d. Basic immunizations are given according to the infant's corrected gestational age.

The nurse is teaching an in-service program to a group of nurses on the topic of gastrointestinal disorders. The nurses in the group make the following statements. Which statement is most accurate related to the diagnosis of gastroesophageal reflux? a. A partial or complete intestinal obstruction occurs. b. A thickened, elongated muscle causes an obstruction at the end of the stomach. c. There are recurrent paroxysmal bouts of abdominal pain. d. In this disorder the sphincter that leads into the stomach is relaxed.

d. In this disorder the sphincter that leads into the stomach is relaxed.

The nurse is caring for a newborn whose mother is HIV positive. The nurse would expect to administer a 6-week course of which medication? a. Lopinavir b. Ritonavir c. Nevirapine d. Zidovudine

d. Zidovudine

When assessing an infant born at 32 weeks' gestation, which finding would lead the nurse to suspect that the newborn has a patent ductus arteriosus (PDA)? a. weak, thready pulse b. decreased pulse rate c. high diastolic arterial pressure d. continuous murmur on auscultation

d. continuous murmur on auscultation

A nurse is teaching a group of parents about characteristics of infants who have failure to thrive. Which of the following characteristics should the nurse include in the teaching? a. intense fear of strangers b. increased risk for childhood obesity c. inability to form close relationships with siblings d. developmental delays

d. developmental delays

A 15-year-old male reports persistent scrotal pain, edema, and nausea since being hit in the groin by a baseball 3 hours ago. Which is the priority action by the nurse? a. applying an ice pack to alleviate the pain b. documenting the swelling and discoloration c. administering pain medications as ordered d. ensuring that the teen is assessed by the physician immediately

d. ensuring that the teen is assessed by the physician immediately

A client asks the nurse what surfactant is. Which explanation would the nurse give as the main role of surfactant in the neonate? a. assists with ciliary body maturation in the upper airways b. helps maintain a rhythmic breathing pattern c. promotes clearing of mucus from the respiratory tract d. helps the lungs remain expanded after the initiation of breathing

d. helps the lungs remain expanded after the initiation of breathing

Compliance

degree of stretch when the heart is filled w blood

Hydrocephalus

excess cerebrospinal fluid in the ventricles or three subarachnoid space -enlarged head diameter treatment: acetazolamide, shunt placement, cath procedure

Opthalmia Neonatorum

eye infection at birth commonly caused by neisseria gonorhoeae or chlamydia -treatment: prophylactic (eyes and thighs to prevent)

Patent Ductus Arteriosus

failure of the ductus arteriosus to close after birth, resulting in an abnormal opening between the pulmonary artery and the aorta -increased pulmonary blood flow -acyanotic -systolic to continuous murmur as the child grows, rales, congestion, increased WOB. severity of the symptoms depends on the amount of blood shunting to the lungs. -treatment: small defect may close on its own. Diuretics (furosemide), indomethacin, ng feeds, surgical closure, increase caloric density of formula or breastmilk

Esophageal Atresia and Tracheoesophageal Fistula

failure of the esophagus to develop as a continuous passage during fetal development. The esophagus ends in a blind pouch; there is a fistula between the distal part of the esophagus and the trachia

Acute Adrenocortical Insufficiency

failure of the organ to work, can be because of long term steroid abuse. Low cortisol. MEDICAL EMERGENCY -extremely low blood pressure, gray appearance, hypoglycemia -treatment: immediate replacement of cortisol, glucose in normal saline solution

IgM immunoglobulin

first responder to pathogenic antigens

B Lymphocytes

form in the bone marrow and release antibodies that fight bacterial infections -antibodies and immunoglobulins

IgD immunoglobulin

found in plasma, binds to b cells

Congenital Adrenal Hyperplasia

genetic disease in which the adrenal gland is overdeveloped, resulting in a deficiency of certain hormones and an overproduction of others treatment: indefinite corticosteroid agent

Growth Hormone Excess

gigantism, acromegaly, big tongue -usually caused by a benign tumor of the anterior pituitary (an ademona) -treatment: GH antagonist

Dysrhythmias: Supraventricular Tachycardia

heart beats greater than 220 bpm in an infant and greater than 160 bpm in a child -treatment: adenosine or synchronized cardioversion

Nursing Intervention for choking for children

heimlich maneuver (abdominal thrusts) -for children that are conscious

Omphalocele

herniation at the umbilicus

Acquired Hypothyroidism (Hashimoto's)

hypothyroidism laster in life -goiter, impaired growth, obesity, lethargy, delayed sexual development, possible nodular thyroid, increased TSH treatment: levothyroxine

Infective endocarditis

inflammation and infection of the endocardium or valves of the heart -persistent low grade fever, malaise, weight loss, arthralgies, symptoms of CHF -treatment: prophylactic antibiotics

Pericarditis

inflammation of the pericardium (sac surrounding the heart) without accumulation of excess pericardial fluid -chest pain worse w inspiration or cough, pericardial friction rub, fever -anti-inflammatories, corticosteroids, colchicine, pericardicentesis to remove fluid

Concussion

injury to the brain caused by a blow -LOC or no LOC dictates imaging -vomiting, headache, confusion upon waking, irritability, amnesia of event -mgmt: decrease stimuli, encourage sleep, wake only once per night immediately post injury to assess LOC

T1DM

insufficient production of insulin -diagnosis: fasting blood glucose, OGTT, A1C (GOLD STANDARD) -POLYURIA, POLYDIPSIA, POLYPHAGIA -MGMT: insulin. monitor blood glucose and urinary ketone levels every 3 hr -call provider if blood glucose is greater than 240mg/dL

T2DM

insulin resistance. later onset -hypertension, acanthosis nigricans (dark discoloration in body fold) -treatment: nutrition and exercise combined w/ oral antiglycemic agent. METFORMIN

Maple Syrup Urine Disease

is caused by a defect in alpha keto acid dehydrogenase, leading to an inability to degrade branched amino acids (isoleucine, leucine, and valine). This illness classically results in poor feeding and dystonia as well asthe maple syrup scent (burnt sugar) in the patients urine after first couple days of life. -treatment: high thiamine and low amino acids (low protein)

Post Term infant

live born infant born after the 42nd week of gestation -complications: high hematocrit and polycythemia, difficulty establishing respiration

Preterm Infant

live born infant born prior to 37 weeks gestation

posttraumatic stress disorder (PTSD)

marked emotional disturbance after experiencing or witnessing a severely stressful event -treated by cognitive behavior therapy, integrated play therapy, family counseling, eye movement desensitization and reprocessing (EMDR), psychological debriefing perhaps through play

IgG immunoglobulin

most abundant activates complement and pivotal in second immune response

Coarctation of the Aorta

narrowing of the aortic ARCH. Compromises blood flow to the lower extremities. It occurs more commonly in males. -obstruction of blood flow -acyanotic -elevated blood pressure to be noted in the right arm (evaluate right arm and then either lower extremity), bounding pulses in the upper extremities, cool skin of lower extremities, weak or absent femoral/lower extremity pulses -treatment: nonsurgical (balloon angioplasty, placement of stents), surgery recommended for infants less than 6 months of age

Anorexia Nervosa

perceive food as revolting and nauseating, refusal to eat or vomit food immediately after eating, possible use of laxative or diuretic or extensively exercising -weight loss, acidosis, dependent edema, hypotension, hypothermia, bradycardia, formation of lanugo, pale, lethargic, amenorrhea

Diaphragmatic Hernia

portion of the abdominal organs, stomach or intestine protrude through the chest wall.

Craniosynostosis

premature closure of the sutures of the skull -surgical intervention

Meningomyelocele

protrusion of the meninges and spinal cord. spina biffida occulta to the max basically

Transient Tachypnea of the Newborn (TTS)

rapid rate of respirations, up to 80bpm when crying. caused by retained lung fluid. -after 5-6 hours, mild retractions, nasal flaring, difficult suck and poor feeding -risks: matenral DM, multifetal pregnancy

Bulimia Nervosa

recurrent and episodic binge eating and purging by vomiting, with awareness eating pattern is abnormal -electrolyte abnormalities; possibly severe erosion of teeth, esophageal tears

Rumination disorder of infancy

report of constantly "spitting up" or vomiting, breath smells sour; signs of failure to thrive

humoral immunity

specific immunity produced by B cells that produce antibodies that circulate in body fluids compliment activation: when activated by antigen-antibody contact, chemotaxis, phagocytosis, and lysis of foreign antigen ensues

Respiratory distress syndrome (RDS)

surfactant deficiency in the lungs. -risk factors: preterm, perinatal asphyxia, maternal DM, PROM, multifetal preg, cold stress, sepsis -tachypnea, nasal flaring, etc.

Serum Sickness

systemic injury initiated by antigen-antibody complexes that circulate in the blood -stiff joints, hives, rash, warmth, and tenderness to the skin

Afterload

the amount of pressure that the heart needs to exert to eject the blood during ventricular contraction

Spina Bifida Occulta

the deformity of the vertebral arch is not apparent from the surface -assess for dimpling, abnormal hair tufts, or discolored skin along spinal tract

S1

the first heart sound, heard when the atrioventricular (mitral and tricuspid) valves close

S2

the second heart sound, heard when the semilunar (aortic and pulmonic) valves close

Ankyloglossia

tongue tied. tight frenulum. can make feeding hard -surgical intervention

Growth Hormone Deficiency

treatment: somatropin (GH replacement) -if pituitary tumor is present, surgery

Sudden Infant Death Syndrome

unexplained death in infancy. > teen moms, closely spaced pregnancies, underweight/premie infants, BPD, twins, native american, economically disadvantaged black infants, narcotic-dependant mom -sleeping on the back is preferred over prone.

Preload

volume of blood found in the ventricles right before they contract. -indicator of circulatory blood volume

LGA infant

weight above 90th percentile common w/ DM paretns

SGA infant

weight below 10th percentile LBW <2500gms VLBW <1500gms ELBW <1000gms

The nurse notices Henry, a 2-month-old with a VSD, is using his accessory muscles while breathing. His mother states he is always breathing like that and sometimes it gets worse when he eats, but she is not concerned because he is not turning blue. What reasons for concern might the nurse identify? (select all that apply) A.Accessory muscle use while breathing B.Accessory muscle use while feeding C.Presence of cyanosis D.Poor weight gain E.Bradycardia

x

When planning the care for a child with Kawasaki disease, which of the following would be most important? A.Making sure he performs postural drainage daily B.Observing him for symptoms of bowel obstruction C.Encouraging him to breathe deeply and cough D.Teaching him to live with a chronic illness

x


Related study sets

Lesson 2: Managing Users and Groups - CompTIA Linux Plus

View Set

Life Insurance Policy (Chapter 4)

View Set

Property Unit 1 - Leasehold Estate/Lease

View Set

Theta Tau, Theta Gamma, Pledge Study Guide

View Set

Applying stem changing verbs (e-ie, o-ue)

View Set

PowerPoint & Other Presentation Options

View Set